PLEASEEE HELPPP
A cone-shaped mold is filled with cake batter. The height of the mold is 15 inches, and the base diameter is 5.5 inches. What is the volume of the mold? Round to the nearest tenth.

Answers

Answer 1

Answer:

118.8 cubic inches

Step-by-step explanation:

volume of a cone is

[tex]vol \: = \frac{1}{3} \times \pi \times \frac{ {d}^{2} }{4} \times h[/tex]

where d is base diameter and

h is cone height.

hence

[tex]vol \: = \frac{1}{3} \times \pi \times \frac{ {5.5}^{2} }{4} \times 15 \\ = \pi \times \frac{30.25}{4} \times 5 \\ = 118.8 \: cu \: in[/tex]

Answer 2

The volume of the cone-shaped mold is 118.73 in³

What is volume?

Volume is the measure of the capacity that an object holds. For example, if a cup can hold 100 ml of water up to the brim, its volume is said to be 100 ml.

Given that a cone shaped mold is filled with the cake batter, the height of the mold is 15 inches, and the base diameter is 5.5,

We need to find the volume of the mold,

Volume of the cone = π × radius² × height / 3

= 3.14 × 2.75² × 15 / 3

= 118.73

Hence, the volume of the cone-shaped mold is 118.73 in³

Learn more about volumes, click;

https://brainly.com/question/1578538

#SPJ3


Related Questions

What is the volume of the cone in the diagram?

Answers

Answer:

471.24

Step-by-step explanation:

V=πr2h/3

radius = 5 and length = 18

answer = 471.24

please mark me brainliest!!

Answer:

The answer Plato wants is 150Pi,

Step-by-step explanation:

Bowl B1 contains one white and two red chips; bowl B2 contains two white and two red chips; bowl B3 contains one white and four red chips. The probabilities of selecting bowl B1, B2, or B3 are 0.3, 0.2, and 0.5, respectively. A bowl is selected using these probabilities and a chip is then drawn at random. Find:

a. P(W), the probability of drawing a white chip.
b. P(B1 Given W), the conditional probability that bowl B1 had been selected, given that a white chip was drawn.

Answers

Answer:

a) 0.3 = 30% probability of drawing a white chip.

b) 0.3333 = 33.33% probability that bowl B1 had been selected, given that a white chip was drawn.

Step-by-step explanation:

Conditional Probability

We use the conditional probability formula to solve this question. It is

[tex]P(B|A) = \frac{P(A \cap B)}{P(A)}[/tex]

In which

P(B|A) is the probability of event B happening, given that A happened.

[tex]P(A \cap B)[/tex] is the probability of both A and B happening.

P(A) is the probability of A happening.

a. P(W), the probability of drawing a white chip.

1/3(one white out of 3) of 0.3(from B1).

1/2(two white out of 4) of 0.2(from B2).

1/5(one white out of 5) of 0.5(from B3). So

[tex]P(W) = 0.3333*0.3 + 0.5*0.2 + 0.2*0.5 = 0.3[/tex]

0.3 = 30% probability of drawing a white chip.

b. P(B1 Given W), the conditional probability that bowl B1 had been selected, given that a white chip was drawn.

The probability of drawing a white chip from B1 is 1/3 out of 0.3, so:

[tex]P(B1 \cap W) = 0.3\frac{1}{3} = 0.1[/tex]

Then the conditional probability is:

[tex]P(B|A) = \frac{P(A \cap B)}{P(A)} = \frac{0.1}{0.3} = 0.3333[/tex]

0.3333 = 33.33% probability that bowl B1 had been selected, given that a white chip was drawn.

If a normal distribution has a mean of 104 and a standard deviation of 6, what is the z-score for a value of 110?

A. 1.5
B. 2
C. 1
D. 0.5

Answers

Answer: 1.5

mean: 104

S.D: 6

110-104/6=1.5

Mason randomly chooses a card from a standard deck of playing cards. What is the probability that it is not an ace, given it is a red card

Answers

Answer:

there are 52 cards in a deck so the chance is a red card (or not a red card) is 26 out of 52.

Step-by-step explanation:

The solution is 12/13

The probability that the selected card is not an ace but a red card is 12/13

What is Probability?

The probability that an event will occur is measured by the ratio of favorable examples to the total number of situations possible

The value of probability lies between 0 and 1

Given data ,

Let the probability of the selected card is not an ace but a red card be P

Now , the equation will be

Mason randomly chooses a card from a standard deck of playing cards

So , the probability that the card is a red card is

There are 26 red cards in the deck of 52 cards

So , the probability of selecting a red card = 26/52

Now , from the 26 red cards , the probability that the card is not an ace would be = 26 - ( Ace of hearts + Ace of diamonds )

And ,

The probability of the selected card is not an ace but a red card P = 24/26

The probability of the selected card is not an ace but a red card P = 12/13

Therefore , the value of P is 12/13

Hence , the probability is 12/13

To learn more about probability click :

https://brainly.com/question/17089724

#SPJ2

5th grade math. correct answer will be marked brainliest

Answers

Answer: How are we going to point it?

Step-by-step explanation:

Find the area of the polygon shown. Enter the number into the box.
m 2
2 mi
10 mi
4 mi
12 mi
1
2

Answers

Answer:

See Explanation

Step-by-step explanation:

The question is incomplete, as the image of the polygon is not given.

I will answer this question with the attached image (similar to your question)

The attached polygon is a trapezoid of the following dimensions.

[tex]Height = 4ft[/tex]

Parallel sides

[tex]Side\ 1 = 4ft[/tex]

[tex]Side\ 2 = 4ft+1ft = 5ft[/tex]

So, the area is:

[tex]Area = \frac{1}{2} * (Side\ 1 + Side\ 2) * Height[/tex]

[tex]Area = \frac{1}{2} * (4ft + 5ft) * 4ft[/tex]

[tex]Area = \frac{1}{2} * 9ft * 4ft[/tex]

[tex]Area = 18ft^2[/tex]

what is the measure and type of zy​

Answers

9514 1404 393

Answer:

  angle zoy is 70°, an acute angle

Step-by-step explanation:

Each of the 18 spaces around the 180° semicircle identifies 10° of arc. There are 7 spaces between rays z and y, so the angle zoy is 70°. It is less than 90°, so it is an acute angle.

Pls Help I will give brianleist

Answers

Answer:

Solution given:

side1=2units

side2=8units

height[h]=6units.

now

area of trapezoid=½h(side1+side2)=½*6(8+2)=

30units².

Find the domain and range of the function y = √x-3 + 6

Answers

Answer:

Domain: [tex][3,\infty)[/tex]

Range: [tex][6,\infty)[/tex]

Step-by-step explanation:

I assume you mean [tex]y=\sqrt{x-3} +6[/tex]?

Take note of how x cannot be less than 3 because it would result in a negative number under the radical, which isn't real. However, x CAN be 3 because [tex]\sqrt{3-3}+6=\sqrt{0}+6=0+6=6[/tex] which is real.

Therefore, the domain of the function is [tex][3,\infty)[/tex]

As for the range of the function, we saw previously that the minimum of the domain resulted in the minimum of the range, which was 6.

Therefore, the range of the function is [tex][6,\infty)[/tex]

See attached graph below for a visual.

Question 21
Find the volume.

Answers

Answer:

C

4712 cm³ exactly

Step-by-step explanation:

10² * pi is the area of the circle

then just multiply with the height

Answer:

C. 1500π ≈ 4710 cm³

Step-by-step explanation:

Volume = πr²h

Volume = π * 100 * 15

Volume = 1500π ≈ 4710 cm³

If my answer is incorrect, pls correct me!

If you like my answer and explanation, mark me as brainliest!

-Chetan K

For the point P(2,-14) and Q(9,-9) find the distance d(P,Q) and the coordinates of the midpoint M of the segment PQ

Answers

Answer:

Step-by-step explanation:

Distance : [tex]\sqrt{(x2-x1)^2 + (y2 - y1)^2} = \sqrt{(9-2)^2 + (-9+14)^2}= \sqrt{49 + 25} = \sqrt{74}[/tex]

Midpoint :

x (m) = (2+9)/2 = 11/2

y (m) = (-14-9)/2 = -23/2

M (11/2, -23/2)

Mr. Suh puts $1,500.00 into savings bonds that pay a simple interest rate of 2.5%. How much money will the bonds be worth at the end of 10 years?

Answers

Answer:

i have no idea

Step-by-step explanation:

5/6+3/9 in the simplest form


HELP PLSS

Answers

Answer:

1 1/6

Step-by-step explanation:

5/6 + 3/9

Simplify 3/9 by dividing the top and bottom by 3

5/6 + 1/3

Get a common denominator of 6

5/6 + 1/3 *2/2

5/6 + 2/6

7/6

Rewriting

6/6 +1/6

1 1/6

5/6 + 1/3 i think
5/6 can’t be simplified but 3/9 can so it’s 1/3 but if you have to add them, use 6 as the common denominator as 3 is a factor then do 1x3 = 3 so it’s simplified to 5/6 + 2/6 = 7/6

find the probability of spinning an A 3 times

Answers

If the probability of spinning an A once is 1/12, then the probability of spinning an A 3 times is 3 × 1/12.

3 × 1/12 = 3/36

Simplify

3/36 = 1/12

The following functions give the populations of four towns with time tt in years.
(i) P=600(1.12)^t
(ii) P=1,000(1.03)^t
(iii) P=200(1.08)^t
(iv) P=900(0.90)^t
Which town has the largest percent growth rate? What is the percent growth rate?

Answers

Solution :

Transforming given equations :

[tex]1)\ P = 600( 1 + \dfrac{12}{100})^t\\\\2)\ P = 1000( 1 + \dfrac{3}{100})^t\\\\3)\ P = 200( 1 + \dfrac{8}{100})^t\\\\4)\ P = 900( 1 - \dfrac{10}{100})^t[/tex]

From above equations we can see that equation 1) has largest percentage growth rate and the percent growth rate is 12% .

Hence, this is the required solution.

Answer: (i), 12%

Step-by-step explanation:

Given

The Population of four towns is given

(i) [tex]P=600(1.12)^t[/tex] i.e.

[tex]\Rightarrow P=600(1+0.12)^t\\\Rightarrow P=600(1+12\%)^t[/tex]

rate=12%

(ii) [tex]P=1000(1.03)^t[/tex]

[tex]\Rightarrow P=1000(1+0.03)^t\\\Rightarrow P=1000(1+3\%)^t[/tex]

rate=3%

(iii) [tex]P=200(1.08)^t[/tex]

[tex]\Rightarrow P=300(1+0.08)^t\\\Rightarrow P=300(1+8\%)^t[/tex]

rate=8%

(iv) [tex]P=900(0.90)^t[/tex]

[tex]\Rightarrow P=900(1-0.10)^t\\\Rightarrow P=900(1-10\%)^t[/tex]

rate=10% fall

So, the largest growth rate is for town (i)  i.e. 12%

i hat is the slope 6/-10 and 3/-13

Answers

Answer:

1

Step-by-step explanation:

Assuming your mean we have the points

(6, -10) and ( 3, -13)

We can use the slope formula

m = (y2-y1)/(x2-x1)

m = ( -13 - -10)/( 3 - 6)

    = (-13+10)/( 3-6)

   = -3 / -3

    = 1

how many liters of a 40% alcohol solution must be mixed with a 65% solution to obtain 50 L of a 50% alcohol solution​

Answers

Step-by-step explanation:

Question 264138: How many liters of a 40% alcohol solution must be mixed with a 65% solution to obtain 20 liters of a 50% solution? x=8 gallons of 65% solution is used. 20-8=12 gallons of 40% solution is used.

please mark as brainliest

What is 10 percent in 66

Answers

6.6 is the answer thanks

You can copy the data into Excel by highlighting the data, right-clicking and selecting Copy, then opening Excel, clicking on a blank cell, and selecting Paste from the Edit menu. Give your answers to at least 3 decimal places accuracy. Keep the data in Excel and keep the answers below handy, as you will need this information again in later questions. 2. Compute the following summary statistics a) The mean for the low income group is: b) The median for the low income group is: c) The standard deviation for the low income group is: d) The mean for the high income group is: e) The median for the high income group is: f) The standard deviation for the high income group is:

Answers

Answer:

Low income group :

Mean = 22.100

Median = 22.000

Standard deviation = 4.717

High Income group :

Mean = 19.771

Median = 20.000

Standard deviation = 3.255

Step-by-step explanation:

Low income : 19,20,13,28,21,27,16,21,19, 19,34,24, 24,14,24,25,23,22,20,21,14,23,17,26,24,24,20,24,27,16,22,17,29,20,25,22,20,34,24,22

High Income : 23 19 19 18 20 18 17 25 13 24 25 17 21 19 17 22 22 15 19 23 19 23 22 20 20 20 13 21 16 13 21 24 23 21 20

Using Excel :

Mean of low income, Σx / n = 884/40 = 22.100

Median of low income group : 1/2(n+1)th term = Median of low income = 22.000

Standard deviation of low income group :

√(Σ(x - mean)²/n) = 4.717

High Income group :

Mean of high income, Σx / n = 692/35 = 19.771

Median of high income group : 1/2(n+1)th term = Median of high income = 20.000

Standard deviation of low income group :

√(Σ(x - mean)²/n) = 3.255

Wei has 150.00 to make a garland using 60-cent balloons

Answers

Answer:

The answer is 0.60b + 0.30w = 150; 60 + 0.30w = 150

Step-by-step explanation:

(2+5i)+(-3-4i) what is the answer to this question?

Answers

Answer:

-1+i

Step-by-step explanation:

(2+5i)+(-3-4i)

=(2-3)+i(5-4)

=-1+i

Answer:

[tex]−1+i[/tex]

Step-by-step explanation:

[tex](2+5i)+(−3−4i)[/tex]

[tex]2+5i−3−4i[/tex]

[tex]−1+5i−4i[/tex]

[tex]−1+i[/tex]

Hope it is helpful.....

Nancy ate 34 ounces of cake. How much cake did she eat?

A. 1.500 lb.
B. 2.125 lb.
C. 2.500 lb.
D. 2.750 lb.

Nancy ate 34 ounces of cake. How much cake did she eat?

A. 1.500 lb.
B. 2.125 lb.
C. 2.500 lb.
D. 2.750 lb.

Answers

Answer:

the answer would be B: 2.125lb.

Answer: B; 2.125lb

Step-by-step explanation: 34 ounces to pounds is 2.13lb (rounded, the exact number is 2.125lb

true or false m angle 1 <m angle 7​

Answers

Answer:

False

Step-by-step explanation:

There might be a chance I am wrong though sorr for the inconvinience

A person takes a multiple-choice exam in which each question has five possible answers. Suppose that the person has no idea about the answers to three of the questions and simply chooses randomly for each one.

Required:
a. What is the probability that the person will answer all three questions correctly?
b. What is the probability that the person will answer exactly two questions correctly?
c. What is the probability that the person will answer exactly one question correctly?
d. What is the probability that the person will answer no questions correctly?
e. Suppose that the person gets one point of credit for each correct answer and that 1/3 point is deducted for each incorrect answer. What is the expected value of the person’s score for the three questions?

Answers

Answer:

a. 0.008 = 0.8% probability that the person will answer all three questions correctly.

b. 0.096 = 9.6% probability that the person will answer exactly two questions correctly.

c. 0.384 = 38.4% probability that the person will answer exactly one question correctly.

d. 0.512 = 51.2% probability that the person will answer no questions correctly.

e. The expected value of the person’s score for the three questions is -0.2.

Step-by-step explanation:

For each question, there are only two possible outcomes. Either the person answers it correctly, or they do not. The probability of a person answering a question correctly is independent of any other question. This means that the binomial probability distribution is used to solve this question.

Binomial probability distribution

The binomial probability is the probability of exactly x successes on n repeated trials, and X can only have two outcomes.

[tex]P(X = x) = C_{n,x}.p^{x}.(1-p)^{n-x}[/tex]

In which [tex]C_{n,x}[/tex] is the number of different combinations of x objects from a set of n elements, given by the following formula.

[tex]C_{n,x} = \frac{n!}{x!(n-x)!}[/tex]

And p is the probability of X happening.

Each question has five possible answers.

Person has no idea which option is correct, so the probability of answering correctly is:

[tex]p = \frac{1}{5} = 0.2[/tex]

Three questions:

This means that [tex]n = 3[/tex]

a. What is the probability that the person will answer all three questions correctly?

This is [tex]P(X = 3)[/tex]. So

[tex]P(X = x) = C_{n,x}.p^{x}.(1-p)^{n-x}[/tex]

[tex]P(X = 3) = C_{3,3}.(0.2)^{3}.(0.8)^{0} = 0.008[/tex]

0.008 = 0.8% probability that the person will answer all three questions correctly.

b. What is the probability that the person will answer exactly two questions correctly?

This is [tex]P(X = 2)[/tex]. So

[tex]P(X = x) = C_{n,x}.p^{x}.(1-p)^{n-x}[/tex]

[tex]P(X = 2) = C_{3,2}.(0.2)^{2}.(0.8)^{1} = 0.096[/tex]

0.096 = 9.6% probability that the person will answer exactly two questions correctly.

c. What is the probability that the person will answer exactly one question correctly?

This is [tex]P(X = 1)[/tex]. So

[tex]P(X = x) = C_{n,x}.p^{x}.(1-p)^{n-x}[/tex]

[tex]P(X = 1) = C_{3,1}.(0.2)^{1}.(0.8)^{2} = 0.384[/tex]

0.384 = 38.4% probability that the person will answer exactly one question correctly.

d. What is the probability that the person will answer no questions correctly?

This is [tex]P(X = 0)[/tex]. So

[tex]P(X = x) = C_{n,x}.p^{x}.(1-p)^{n-x}[/tex]

[tex]P(X = 0) = C_{3,0}.(0.2)^{0}.(0.8)^{3} = 0.512[/tex]

0.512 = 51.2% probability that the person will answer no questions correctly.

e. Suppose that the person gets one point of credit for each correct answer and that 1/3 point is deducted for each incorrect answer. What is the expected value of the person’s score for the three questions?

The expected number of correct answer is:

[tex]E(X) = np = 3*0.2 = 0.6[/tex]

And the expected number of wrong answers is 3 - 0.6 = 2.4. So, the expected score is:

[tex]S(x) = 0.6 - \frac{2.4}{3} = 0.6 - 0.8 = -0.2[/tex]

The expected value of the person’s score for the three questions is -0.2.

please answer both questions thanks ​

Answers

Answer:

1. tan(65) = x/3

x = 3tan(65)

= 6.434

≈ 6.4

(nearest 10th)

2. cos(43) = AB/17

AB = 17cos(43)

= 12.433

≈ 12.43

(nearest 100th)

what does this function show at x=1?
A. removable discontinuity
B. Jump discontinuity
C. Infinite discontinuity
D. Continuity

Answers

Answer:

c

Step-by-step explanation:

yeah tha answer is c I can't explain it rn but for now put c

In Triangle QRS, Angle R is a right angle, m angle Q = 57° and QS = 35. Which of the following expressions correctly
calculates the length of line QR ?

Answers

9514 1404 393

Answer:

  (3)  35·cos(57°)

Step-by-step explanation:

The relevant trig relation is ...

  Cos = Adjacent/Hypotenuse

For the given triangle, this is ...

  cos(57°) = QR/QS

Multiplying by QS gives ...

  QS·cos(57°) = QR

Filling in the value of QS, we have ...

  QR = 35·cos(57°)

Answer:

Option (3) 35 cos 57°

Explanation:

Hope it helps you!!!!v_v

What’s the ara of each figure?

Answers

Answer:

433 in.²

Step-by-step explanation:

Divide the figure into 2 rectangles

Area of the figure = area of the bigger rectangle + area of smaller rectangle

✔️Area of bigger rectangle = L*W

L = 29 in.

W = 13 in.

Area of the bigger rectangle = 29*13

= 377 in.²

✔️Area of the smaller rectangle = L*W

L = 7 in.

W = 8 in.

Area of the bigger rectangle = 7*8

= 56 in.²

✅Area of the figure = 377 + 56 = 433 in.²

pls help I need a good grade

Answers

Answer:

D

Step-by-step explanation:

because in standard form the first number can not be greater than 10 above 10 it should be in points. e.g. D must be 6.12×10^5

Amy has a bag with 2 pink marbles and 3 purple marbles. She picks two marbles. How much greater is the probability of Amy picking two consecutive pink marbles if she replaces her first marble as opposed to her leaving the first marble she picks out of the bag?

Answers

Answer:

2/5 is the answer bc there are 5 total marbles and you have more purple marbles then pink ones so the answer is 2/5 or 40%

Other Questions
A pack of six cans of coffee cost $12. How much would 15 cans ofcoffee cost?Answer $30 Bc 6can = $12If 1can =2$So 15 cans =30$ What is the result of subtracting the second equation from the first Rebecca walks nine dogs each week she gets paid $10 a week +2 for each dog she walks which expression can be used to find how much is Rebecca gets paid in one week Pls help school almost over I need to pass Solve for x. A. x = 12 B. x = 4 C. x = 3 D. x = 1 Solve.5. The graph of f(x)= x is reflected across the x-axis. The graph is thentranslated 11 units up and 7 units to the left. Write the equation of thetransformed function. A Michelson interferometer operating at a 400 nm wavelength has a 3.70-cm-long glass cell in one arm. To begin, the air is pumped out of the cell and mirror M2 is adjusted to produce a bright spot at the center of the interference pattern. Then a valve is opened and air is slowly admitted into the cell. The index of refraction of air at 1.00 atm pressure is 1.00028.How many bright-dark-bright fringe shifts are observed as the cell fills with air? Why less rainfall and hotter weather would affect us?? Help me plsss If any one could please help Read the following passage from "Joining Forces for Freedom." Slect thethree words or phrases that indicate the text is historical fiction."The year was 1918, and Emma lived in Buffalo, New York. One day inApril almost a year before, her dad came home from work with a newspaperunder his arm. The headline said, 'PRESIDENT CALLS FOR WARDECLARATION.' Dad explained that Germany was trying to take over othercountries and attacking ships, including American ships. So PresidentWilson had asked Congress to declare war and join forces with Germany'senemies. America had a duty to fight for freedom." Why did the Spanish want to convert American peoples to Christianity? How did the Spanish try to convert the American Indians? the number 32747 written to 4 significant figure is does salt make water boil faster? For the given function, find the vertical and horizontal asymptote(s) (if there are any).f(x) = (x-5)/(x^2-1) Which diagram arranges the types of business organizations from the mostcomplicated to start to the least complicated to start?A. Corporation - Partnership -Sole proprietorshipB. Sole proprietorship - Partnership - CorporationC. Partnership - Sole proprietorship - CorporationD. Corporation - Sole proprietorship - Partnership How many molecules of water vapor participate in the above reaction if 90 g of water vapor are available? people speak English all over the world passive How does the difference between the characters spoken words and internial thought affect the story In which number does the digit 9 have a value that is ten times greater than the value of the digit 9 in the number 593,104? 95,378 529,092 691,525 922,198 There is a bag with only red marbles and blue marbles.The probability of randomly choosing a blue marble is 78.There are 56 marbles in total in the bag and each is equally likely to be chosen.Work out how many red marbles there must be.